Stay ahead of learning milestones! Enroll in a class over the summer!

G
Topic
First Poster
Last Poster
Hard geometry
Lukariman   7
N 4 minutes ago by Captainscrubz
Given circle (O) and chord AB with different diameters. The tangents of circle (O) at A and B intersect at point P. On the small arc AB, take point C so that triangle CAB is not isosceles. The lines CA and BP intersect at D, BC and AP intersect at E. Prove that the centers of the circles circumscribing triangles ACE, BCD and OPC are collinear.
7 replies
Lukariman
May 14, 2025
Captainscrubz
4 minutes ago
Prove that the triangle is isosceles.
TUAN2k8   0
5 minutes ago
Source: My book
Given acute triangle $ABC$ with two altitudes $CF$ and $BE$.Let $D$ be the point on the line $CF$ such that $DB \perp BC$.The lines $AD$ and $EF$ intersect at point $X$, and $Y$ is the point on segment $BX$ such that $CY \perp BY$.Suppose that $CF$ bisects $BE$.Prove that triangle $ACY$ is isosceles.
0 replies
TUAN2k8
5 minutes ago
0 replies
Simple but hard
Lukariman   2
N 30 minutes ago by Lukariman
Given triangle ABC. Outside the triangle, construct rectangles ACDE and BCFG with equal areas. Let M be the midpoint of DF. Prove that CM passes through the center of the circle circumscribing triangle ABC.
2 replies
Lukariman
Today at 2:47 AM
Lukariman
30 minutes ago
D1033 : A problem of probability for dominoes 3*1
Dattier   1
N an hour ago by Dattier
Source: les dattes à Dattier
Let $G$ a grid of 9*9, we choose a little square in $G$ of this grid three times, we can choose three times the same.

What the probability of cover with 3*1 dominoes this grid removed by theses little squares (one, two or three) ?
1 reply
Dattier
Yesterday at 12:29 PM
Dattier
an hour ago
Please I need help
yaybanana   2
N an hour ago by yaybanana
Source: Samin Riasat Handout
Please can someone help me, I'm bad at inequalities and I have no clue on how to solve this :

Let $a,b,c$ be positive reals, s.t $a+b+c=1$, prove that :

$\frac{a}{\sqrt{a+2b}}+\frac{b}{\sqrt{b+2c}}+\frac{c}{\sqrt{c+2a}}<\sqrt{\frac{3}{2}}$
2 replies
yaybanana
an hour ago
yaybanana
an hour ago
LCM genius problem from our favorite author
MS_Kekas   2
N an hour ago by AshAuktober
Source: Kyiv City MO 2022 Round 2, Problem 8.1
Find all triples $(a, b, c)$ of positive integers for which $a + [a, b] = b + [b, c] = c + [c, a]$.

Here $[a, b]$ denotes the least common multiple of integers $a, b$.

(Proposed by Mykhailo Shtandenko)
2 replies
MS_Kekas
Jan 30, 2022
AshAuktober
an hour ago
How many residues modulo p are sums of two squares?
Tintarn   8
N an hour ago by thaiquan2008
Source: Austrian MO 2024, Final Round P6
For each prime number $p$, determine the number of residue classes modulo $p$ which can
be represented as $a^2+b^2$ modulo $p$, where $a$ and $b$ are arbitrary integers.

(Daniel Holmes)
8 replies
Tintarn
Jun 1, 2024
thaiquan2008
an hour ago
Interesting inequalities
sqing   2
N an hour ago by sqing
Source: Own
Let $a,b,c \geq 0 $ and $ab+bc+ca- abc =3.$ Show that
$$a+k(b+c)\geq 2\sqrt{3 k}$$Where $ k\geq 1. $
Let $a,b,c \geq 0 $ and $2(ab+bc+ca)- abc =31.$ Show that
$$a+k(b+c)\geq \sqrt{62k}$$Where $ k\geq 1. $
2 replies
sqing
5 hours ago
sqing
an hour ago
abc = 1 Inequality generalisation
CHESSR1DER   7
N an hour ago by CHESSR1DER
Source: Own
Let $a,b,c > 0$, $abc=1$.
Find min $ \frac{1}{a^m(bx+cy)^n} + \frac{1}{b^m(cx+ay)^n} + \frac{1}{c^m(cx+ay)^n}$.
$1)$ $m,n,x,y$ are fixed positive integers.
$2)$ $m,n,x,y$ are fixed positive real numbers.
7 replies
CHESSR1DER
Yesterday at 6:40 PM
CHESSR1DER
an hour ago
Old problem
kwin   2
N an hour ago by lbh_qys
Let $ a, b, c > 0$ . Prove that:
$$(a^2+b^2)(b^2+c^2)(c^2+a^2)(ab+bc+ca)^2 \ge 8(abc)^2(a^2+b^2+c^2)^2$$
2 replies
kwin
Today at 3:44 AM
lbh_qys
an hour ago
IMO Solution mistake
CHESSR1DER   0
2 hours ago
Source: Mistake in IMO 1982/1 4th solution
I found a mistake in 4th solution at IMO 1982/1. It gives answer $660$ and $661$. But right answer is only $660$. Should it be reported somewhere in Aops?
0 replies
CHESSR1DER
2 hours ago
0 replies
All the numbers to be zero after finitely many operations
orl   9
N 3 hours ago by User210790
Source: IMO Shortlist 1989, Problem 19, ILL 64
A natural number is written in each square of an $ m \times n$ chess board. The allowed move is to add an integer $ k$ to each of two adjacent numbers in such a way that non-negative numbers are obtained. (Two squares are adjacent if they have a common side.) Find a necessary and sufficient condition for it to be possible for all the numbers to be zero after finitely many operations.
9 replies
orl
Sep 18, 2008
User210790
3 hours ago
A number theory problem
super1978   2
N 4 hours ago by Tintarn
Source: Somewhere
Let $a,b,n$ be positive integers such that $\sqrt[n]{a}+\sqrt[n]{b}$ is an integer. Prove that $a,b$ are both the $n$th power of $2$ positive integers.
2 replies
super1978
May 11, 2025
Tintarn
4 hours ago
Medium geometry with AH diameter circle
v_Enhance   95
N May 11, 2025 by Markas
Source: USA TSTST 2016 Problem 2, by Evan Chen
Let $ABC$ be a scalene triangle with orthocenter $H$ and circumcenter $O$. Denote by $M$, $N$ the midpoints of $\overline{AH}$, $\overline{BC}$. Suppose the circle $\gamma$ with diameter $\overline{AH}$ meets the circumcircle of $ABC$ at $G \neq A$, and meets line $AN$ at a point $Q \neq A$. The tangent to $\gamma$ at $G$ meets line $OM$ at $P$. Show that the circumcircles of $\triangle GNQ$ and $\triangle MBC$ intersect at a point $T$ on $\overline{PN}$.

Proposed by Evan Chen
95 replies
v_Enhance
Jun 28, 2016
Markas
May 11, 2025
Medium geometry with AH diameter circle
G H J
Source: USA TSTST 2016 Problem 2, by Evan Chen
The post below has been deleted. Click to close.
This post has been deleted. Click here to see post.
v_Enhance
6877 posts
#1 • 25 Y
Y by doxuanlong15052000, john111111, anantmudgal09, Davi-8191, AlastorMoody, Vietjung, rashah76, itslumi, v4913, tenebrine, sotpidot, HWenslawski, innout, anonman, megarnie, HamstPan38825, Jc426, Lamboreghini, CyclicISLscelesTrapezoid, tiendung2006, mathmax12, Adventure10, Mango247, geobo, Rounak_iitr
Let $ABC$ be a scalene triangle with orthocenter $H$ and circumcenter $O$. Denote by $M$, $N$ the midpoints of $\overline{AH}$, $\overline{BC}$. Suppose the circle $\gamma$ with diameter $\overline{AH}$ meets the circumcircle of $ABC$ at $G \neq A$, and meets line $AN$ at a point $Q \neq A$. The tangent to $\gamma$ at $G$ meets line $OM$ at $P$. Show that the circumcircles of $\triangle GNQ$ and $\triangle MBC$ intersect at a point $T$ on $\overline{PN}$.

Proposed by Evan Chen
Z K Y
The post below has been deleted. Click to close.
This post has been deleted. Click here to see post.
v_Enhance
6877 posts
#2 • 25 Y
Y by anantmudgal09, jlammy, math4444, baladin, Davi-8191, Iyerie, Anar24, AlastorMoody, Omeredip, amar_04, mathlogician, v4913, tenebrine, sotpidot, TheCollatzConjecture, innout, megarnie, hsiangshen, HamstPan38825, Jc426, Lamboreghini, mathmax12, EpicBird08, Adventure10, Mango247
Denote by $\triangle DEF$ the orthic triangle. Note that $\overline{AG}$, $\overline{EF}$, $\overline{BC}$ are concurrent at $R$ by radical axis, and that $\overline{PA}$ and $\overline{PG}$ are tangents to $\gamma$.

Now, consider circles $(PAGM)$, $(MFDNE)$, and $(MBC)$. They intersect at $M$ but have radical center $R$, so are coaxial; assume they meet again at $T \in \overline{RM}$, say. Then $\angle PTM$ and $\angle MTN$ are both right angles, hence $T$ lies on $\overline{PN}$.

Finally $H$ is the orthocenter of $\triangle ARN$, and thus the circle with diameter $\overline{RN}$ passes through $G$, $Q$, $N$.
This post has been edited 2 times. Last edited by v_Enhance, Jun 29, 2016, 6:48 AM
Reason: Un-hide now that people have had chance to try it
Z K Y
The post below has been deleted. Click to close.
This post has been deleted. Click here to see post.
LeVietAn
375 posts
#3 • 3 Y
Y by AllanTian, Adventure10, Mango247
My solutions:
We have $\overline{G, H, N}$ (well-known). Let $AG\cap BC=S, AH\cap BC=D$, and let $MK\perp PN at K$.
We have $AG, KM, DN$ are concurent at point $S$ is the radical center of ${\odot (APGKM), \odot (MKDN), \odot (AGDN)}$.
Becaues $AH\perp SN, NH\perp AS$ $\Rightarrow H$ is othorcenter of $\triangle ANS$ $\Rightarrow Q\in SH$.
We have $SK.SM=SG.SA=SB.SC$ $\Rightarrow K\in \odot (BCM)$.
We have $K\in \odot (SN)\equiv (GQN)$.
So, $(GNQ)\cap (MBC)\cap PN=K$. DONE
Z K Y
The post below has been deleted. Click to close.
This post has been deleted. Click here to see post.
ABCDE
1963 posts
#4 • 4 Y
Y by char2539, amuthup, Adventure10, Mango247
I always overcomplicate Evan's medium geometry proposals lol

Click to reveal hidden text
Z K Y
The post below has been deleted. Click to close.
This post has been deleted. Click here to see post.
houssam9990
33 posts
#6 • 2 Y
Y by Adventure10, Mango247
Click to reveal hidden text
This post has been edited 1 time. Last edited by houssam9990, Jun 28, 2016, 4:21 PM
Z K Y
The post below has been deleted. Click to close.
This post has been deleted. Click here to see post.
ThE-dArK-lOrD
4071 posts
#7 • 3 Y
Y by talkon, Adventure10, Mango247
Let $U,V$ be the feet of altitudes from $B,C$ respectively.
Let $UV\cap BC=X$ and let $XM \cap \mathcal{N} =T$ where $\mathcal{N}$ is the nine-point circle of $\triangle{ABC}$.
Considering radical axes of $\mathcal{N},(ABC),(MBC)$ we found that $X$ is the radical center and so $T \in (MBC)$.
Considering those of $(PAG),(ABC),(MBC)$ we found that $X$ is the radical center and so $T\in (PAG)$.
Then, we have $\angle{PTM}=90^{\circ}$ and $\angle{NTM}=\angle{NUM}=90^{\circ}$. So $T\in PN$.
From $\angle{XTN}=90^{\circ}=\angle{XQN}=\angle{XGN}=90^{\circ}$ we get $T\in (GQN)$.
Hence $(GQN)\cap(MBC)=T\in PN$.
This post has been edited 4 times. Last edited by ThE-dArK-lOrD, Jun 13, 2019, 6:04 AM
Z K Y
The post below has been deleted. Click to close.
This post has been deleted. Click here to see post.
talkon
276 posts
#8 • 3 Y
Y by ThE-dArK-lOrD, Adventure10, Mango247
My solution.
This post has been edited 3 times. Last edited by talkon, Jun 28, 2016, 5:36 PM
Z K Y
The post below has been deleted. Click to close.
This post has been deleted. Click here to see post.
liberator
95 posts
#9 • 12 Y
Y by abk2015, v_Enhance, PARISsaintGERMAIN, BobaFett101, JasperL, rashah76, parola, Abidabi, sotpidot, ike.chen, Adventure10, Mango247
[asy]
unitsize(3.5cm);
pointpen=black; pathpen=rgb(0.4,0.6,0.8);
pointfontpen=fontsize(10);

path carc(pair A, pair B, pair C, real d=0, bool dir=CW) {
pair O=circumcenter(A,B,C);
return arc(O,circumradius(A,B,C),degrees(A-O)+d,degrees(C-O)-d,dir);
}

pair A=dir(120), B=dir(205), C=dir(-25), E=foot(B,C,A), F=foot(C,A,B), H=A+B+C, O=(0,0), M=(A+H)/2, N=(B+C)/2, R=extension(E,F,B,C), G=foot(N,A,R), P=extension(O,M,A,A+C-B), Q=extension(R,H,A,N), T=foot(M,P,N);

D(unitcircle,heavygreen);
DPA(CP(M,A)^^A--P--G,red);
D(carc(R,Q,N),purple);
D(carc(B,M,C),orange);
D(A--B--C--cycle,1);
DPA(A--R--E^^B--R--M^^P--N);
DPA(H--A--N^^P--O,dashed+pathpen);

D("A",A,dir(A));
D("B",B,SW);
D("C",C,SE);
D("E",E,dir(0));
D("F",F,dir(F));
D("G",G,NW);
D("H",H);
D("M",M,NE);
D("N",N);
D("O",O);
D("P",P,dir(P));
D("Q",Q);
D("R",R,dir(R));
D("T",T);
[/asy]
Let $\triangle DEF$ be the orthic triangle, and note that $AG,EF,BC$ are concurrent at $R$ by radical axis. Let $T=MR\cap PN$. With respect to $\gamma$, $\overline{AG}$ is the polar of $P$ and $\overline{EF}$ is the polar of $N$, thus $\overline{PN}$ is the polar of $R$ (La Hire), so $R,T$ are inverses, and $\angle RTN=90^{\circ}$.

$H$ is the orthocenter of $\triangle ARN$ (Brokard), and thus the circle on diameter $\overline{RN}$ passes through $G,T,Q,N$. But also $RT\cdot RM=RE\cdot RF=RB\cdot RC$, so $T$ lies on $(MBC)$, and the result follows.

Comment. This is a nice, novel problem based off a popular configuration. Well done Evan!
This post has been edited 1 time. Last edited by liberator, Jun 28, 2016, 5:37 PM
Reason: added comment
Z K Y
The post below has been deleted. Click to close.
This post has been deleted. Click here to see post.
EulerMacaroni
851 posts
#10 • 2 Y
Y by Adventure10, Mango247
Let $DEF$ be the orthic triangle, $V$ be the harmonic conjugate of $D$ with respect to segment $\overline{BC}$, and $T'$ be the harmonic conjugate of $D$ with respect to $EF$ in the nine-point circle. Projecting from $M$ onto line $EF$ implies that $M,T',V$ are collinear, so $$VT'\cdot VM=VF\cdot VE=VB\cdot VC$$so $T'\in\odot(MBC)$, and since $\angle MT'N=\pi-\angle VT'N=\frac{\pi}{2}$, $T'\in \odot(GQN)$, hence $T'\equiv T$. Finally, $$VT\cdot VM=VB\cdot VC=VG\cdot VA$$so pentagon $APGTM$ is cyclic, and $\angle PTM=\pi-\angle PAM=\frac{\pi}{2}$.
Z K Y
The post below has been deleted. Click to close.
This post has been deleted. Click here to see post.
First
2352 posts
#11 • 1 Y
Y by Adventure10
V_Enhance, how did you come up with this problem?
Z K Y
The post below has been deleted. Click to close.
This post has been deleted. Click here to see post.
PROF65
2016 posts
#12 • 2 Y
Y by Adventure10, Mango247
Let $ S= AG\cap BC $ we know that $NG\cap SQ=H$ is the orthocenter of $ASN$ if J is the center of $NSGQ$ then $\widehat{JGM}=\widehat{GNS}=\widehat{GQS}=\widehat{GQH} $ thus $GJ$ is tangent to $(M)$ i.e. $\overline{JGP} \perp GM $ besides $   OM \perp AG,NG\perp AG $ then $OM \parallel NG$ implies $ \widehat{GPM}=\widehat{JGN}=\widehat{GQH}=\widehat{GAH}    =\widehat{GAM} $ hence $GPMA$ s cyclic but $GJ\perp GM$ thus $PA \perp AM=AH$ implies $PA\parallel BC $ , applying the converse of Reim's to $N-R-P , S-G-A$ where $ R=NP\cap (GMAP) $ yields $ GRSB$ is cyclic .In the other hand $RM\perp PR, SR\perp NR $ we deduce $S,R,M $ are collinear ; further $GA,RM,BC$ are concurrent so by radical axis 's we conclude $RMBC$ is cyclic.
R HAS.
Z K Y
The post below has been deleted. Click to close.
This post has been deleted. Click here to see post.
DrMath
2130 posts
#13 • 2 Y
Y by Adventure10, Mango247
Nice problem!

Let $\triangle DEF$ be the orthic triangle. First, $\angle AQH=\angle AQA' = 90$ where $A'$ is the antipode of $A$. Thus $Q, H, N, A'$ are collinear. An easy angle chase gives $\angle MFN =\angle MEN =90$, so $QH$, the tangent to $(AEF)$ at $E$, and the tangent at $F$ concur. Thus $QFHE$ is harmonic. A perspectivity at $A$ gives $XBDC$ harmonic where $X=AQ\cap BC$. Thus $X\in EF$ by a well known lemma. On the other hand, since $NH\perp AQ$ and $AH\perp XN$, $XH\perp AG$ and $X, H, G$ are collinear. Moreover, $(XQGN)$ is cyclic.

Let $T$ be the intersection of circles $(XQGN)$ and $(MFDNE)$ other than $N$. Note $XT\cdot XM = XF\cdot XE=XB\cdot XC$, so $T$ lies on $(MBC)$. By Radical Axis on $(AQFHGE), (XQGN), (MFDNE)$, $GQ, EF, TN$ concur. Let $R$ be the point of concurrence. On the other hand, by Radical Axis on $(MQDG), (XQGN), (MFDNE)$, we have $MD, GQ, TN$ concur. Thus $R$ lies on $AH$ as well. We claim that $P, R, N$ are collinear.

Note $MO\parallel QHN\perp QA$, so $PM\perp QA$. Thus $PQ=PA$, so $P$ is the intersection of the tangents to $(AQFHGE)$ at $A, Q$. But recall $N$ was the intersection of the tangents at $E, F$. Thus by Pascal's on $AQHQAG$ we deduce $P, R, N$ are collinear, as desired. $\square$
This post has been edited 4 times. Last edited by DrMath, Jun 29, 2016, 5:15 PM
Z K Y
The post below has been deleted. Click to close.
This post has been deleted. Click here to see post.
MS_Kekas
275 posts
#14 • 4 Y
Y by Mindstormer, Didier, Adventure10, Mango247
Niche ne ponel no pooral
Z K Y
The post below has been deleted. Click to close.
This post has been deleted. Click here to see post.
v_Enhance
6877 posts
#15 • 8 Y
Y by niraekjs, smy2012, AlastorMoody, v4913, HamstPan38825, Adventure10, Mango247, Rounak_iitr
First wrote:
V_Enhance, how did you come up with this problem?
Well, the original proposal was to show that the nine-point circle and $(MBC)$ intersected on line $PN$. The points $G$ and $Q$ were added after the fact, to prevent people from using straight coordinates.

Here is also my original solution: Let $L$ be diametrically opposite $A$ on the circumcircle. Denote by $\triangle DEF$ the orthic triangle. Let $X = \overline{AH} \cap \overline{EF}$. Finally, let $T$ be the second intersection of $(MFDNE)$ and $(MBC)$.

We begin with a few easy observations. First, points $H$, $G$, $N$, $L$ are collinear and $\angle AGL = 90^\circ$. Also, $Q$ is the foot from $H$ to $\overline{AN}$. Consequently, lines $AG$, $EF$, $HQ$, $BC$, $TM$ concur at a point $R$ (radical axis). Moreover, we already know $\angle MTN = 90^\circ$. This implies $T$ lies on the circle with diameter $\overline{RN}$, which is exactly the circumcircle of $\triangle GQN$.

Note by Brokard's Theorem on $AFHE$, the point $X$ is the orthocenter of $\triangle MBC$. But $\angle MTN = 90^\circ$ already, and $N$ is the midpoint of $\overline{BC}$. Consequently, points $T$, $X$, $N$ are collinear.

Finally, we claim $P$, $X$, $N$ are collinear, which solves the problem. Note $P = \overline{GG} \cap \overline{AA}$. Set $K = \overline{HNL} \cap \overline{AP}$. Then by noting \[ -1 = (D,X;A,H) \overset{N}{=} (\infty, \overline{NX} \cap \overline{AK}; A, K) \]we see that $\overline{NX}$ bisects segment $\overline{AK}$, as desired. (A more projective finish is to show that $\overline{PXN}$ is the polar of $R$ to $\gamma$).
Attachments:
Z K Y
The post below has been deleted. Click to close.
This post has been deleted. Click here to see post.
Aiscrim
409 posts
#17 • 1 Y
Y by Adventure10
Let $A^\star$ be the antipode of $A$ in $(ABC)$ and let $QN\cap (ABC)=\{A,Q^\star\}$. It is well known that $A^\star$ is the reflection of $H$ over $N$. As $HQ\parallel Q^\star A^\star$ (they are both perpendicular to $AN$) and $NH=NA^\star$, we infer that $HQA^\star Q^\star$ is a parallelogram, so $NQ=NQ^\star$. Also note that $G-H-N$ are collinear.

Let $\{T\}=AG\cap HQ,\ \{R\}=AH\cap GQ$. Then $T$ lies on the polar of $P$ wrt $\gamma$, so $P$ lies on the polar of $T$ wrt to $\gamma$, i.e. $P\in NR$. Observe that by Power of Point we have
$$ NG\cdot NH=NA\cdot NQ=NA\cdot NQ^\star=NB\cdot NC$$
The inversion $\mathcal{I} (N,NB^2)$ swaps $G$ with $H$, $Q$ with $A$, and leaves $B$ and $C$ invariant. Let $\mathcal{I}(M)=M^\star$. Under inversion, the concurrency of $(NGQ),(MBC)$ and $PN$ is equivalent to the concurrency of $AH,(M^\star BC)$ and $NP$, i.e. to proving that $R \in (M^\star BC)$.

A little computation shows us that $BR\perp MC$, whence $R$ is the orthocenter of $\triangle{MBC}$, so $$\widehat{BRC}=180^\circ-\widehat{BMC}=\widehat{BM^\star C}\Rightarrow R \in (M^\star BC)$$
Z K Y
G
H
=
a